Difference between revisions of "1973 Canadian MO Problems/Problem 7"

m
m
Line 2: Line 2:
  
 
<math>\text{(i)}</math> Observe that
 
<math>\text{(i)}</math> Observe that
<math>\frac{1}{1}=</math> <math>\frac{1}{2}+</math> <math>\frac{1}{2};</math> <math>\quad</math> <math>\frac{1}{2}=\frac{1}{3}+\frac{1}{6};\quad \frac{1}{3}=\frac{1}{4}+\frac{1}{12};\qu...</math>
+
<math>\frac{1}{1}=</math> <math>\frac{1}{2}+</math> <math>\frac{1}{2};</math> <math>\quad</math> <math>\frac{1}{2}=</math> <math>\frac{1}{3}+</math> <math>\frac{1}{6};</math> <math>\quad \frac{1}{3}=</math> <math>\frac{1}{4}+\frac{1}{12};\quad...</math>
 
State a general law suggested by these examples, and prove it.
 
State a general law suggested by these examples, and prove it.
  

Revision as of 20:06, 4 December 2015

Problem

$\text{(i)}$ Observe that $\frac{1}{1}=$ $\frac{1}{2}+$ $\frac{1}{2};$ $\quad$ $\frac{1}{2}=$ $\frac{1}{3}+$ $\frac{1}{6};$ $\quad \frac{1}{3}=$ $\frac{1}{4}+\frac{1}{12};\quad...$ State a general law suggested by these examples, and prove it.


$\text{(ii)}$ Prove that for any integer $n$ greater than $1$ there exist positive integers $i$ and $j$ such that $\frac{1}{n}= \frac{1}{i(i+1)}+\frac{1}{(i+1)(i+2)}+\frac{1}{(i+2)(i+3)}+\cdots+\frac{1}{j(j+1)}.$

Solution

$\text{(i)}$ We see that:

$\frac{1}{n(n+1)}+\frac{1}{n+1} = \frac{1}{n}$

We prove this by induction. Let $P(n)=\frac{1}{n(n+1)}+\frac{1}{n+1}= \frac{1}{n}.$ Base case: $n = 1 \Rightarrow P(1):\text{LHS} = 1, \text{while RHS} = \frac{1}{1} = 1.$ Therefore, $P(1)$ is true. Now, assume that $P(n)$ is true for some $n=k$. Then:

$\begin{matrix} \text{LHS of } P(k+1) &=& \frac{1}{(k+1)((k+1)+1)}+\frac{1}{(k+1)+1}\\ &=& \frac{1}{(k+1)(k+2)}+\frac{1}{k+2}\\ &=& \frac{1}{(k+1)(k+2)}+\frac{k+1}{(k+2)(k+1)}\\ &=& \frac{k+2}{(k+1)(k+2)}\\  &=& \frac{1}{k+1}  &=& \text{RHS of } P(k+1)\\\end{matrix}$

Thus, by induction, the formula holds for all $n \in \mathbb{Z^{+}}$

$\text{(ii)}$ Incomplete

See also

1973 Canadian MO (Problems)
Preceded by
Problem 6
1 2 3 4 5 Followed by
Problem 1